Тёмный

The Reciprocal Prime Series (this proof should be taught in calculus!) 

Dr. Trefor Bazett
Подписаться 424 тыс.
Просмотров 114 тыс.
50% 1

Опубликовано:

 

10 сен 2024

Поделиться:

Ссылка:

Скачать:

Готовим ссылку...

Добавить в:

Мой плейлист
Посмотреть позже
Комментарии : 228   
@alexismiller2349
@alexismiller2349 Год назад
One line proof by contradiction using probabilities: If the sum of 1/p_i converged, the value of the sum would have been a pretty famous constant thus I probably would have heard of it by now
@cufflink44
@cufflink44 Год назад
Full marks! 😂
@DevinBaillie
@DevinBaillie Год назад
But you neglect the case where it converges to an already famous constant. Nobody would take note if it was just another formula that evaluates to some multiple of pi, for example.
@alexismiller2349
@alexismiller2349 Год назад
@@DevinBaillie true, therefore one needs to calculate the terms of the series until it exceeds 7, which would conclude since there are no famous constants bigger then 7
@DevinBaillie
@DevinBaillie Год назад
@@alexismiller2349 it could still work out to n×pi for some n.
@alexismiller2349
@alexismiller2349 Год назад
@@DevinBaillie I've considered your criticism and so I've decided to rescind my article to the Field's institute in light of your contribution
@kasuha
@kasuha Год назад
The proof that the sum at 7:47 diverges can be made simpler by realizing that 1/(1+N) > 1/2N for any N>2 so the terms of the sum can be compared 1:1 with harmonic series divided by constant 2*P1*...*PN. And since infinity divided by any finite number is still infinity, the series must diverge. No limit necessary.
@mushyomens6885
@mushyomens6885 Год назад
can you elaborate how 1/2N is a harmonic series divided by a constant when the N gets multipled by a new prime for each subsequent term?
@Jack_Callcott_AU
@Jack_Callcott_AU Год назад
@@mushyomens6885 The proof by @kasuha is valid and a bit simpler than the one in the video. The harmonic series is Σ 1/N which is known to diverge. We can always factor out a constant from an infinite sum so Σ 1/(2*N) = 1/2*Σ1/N is 1/2 *( a divergent series) which, of course, also diverges. I hope this has explained the problem for you.👍
@ronald3836
@ronald3836 10 месяцев назад
Alternatively, use 1+ i * p_1*...*p_N = C * sum 1/(1+i), which obviously diverges.
@johnchessant3012
@johnchessant3012 Год назад
Very neat! Euler's original proof for this is also pretty cool, he recalls the factorization of the harmonic series, namely the product over primes of 1/(1 - 1/p), and takes the log of both sides to get that the sum over primes of log(1/(1 - 1/p)) diverges. He then Taylor expands this to get the sum over primes of 1/p + 1/(2p^2) + 1/(3p^3) + ..., where the sums of 1/(2p^2), 1/(3p^3), etc. are subseries of 1/(2n^2), 1/(3n^3), ..., which converge. Thus for this to diverge it must be the case that the sum of 1/p diverges.
@DrTrefor
@DrTrefor Год назад
I do love this proof too and it does a better job than the one I should at illustrating the log(log(x)) divergence
@sharpnova2
@sharpnova2 Год назад
@@DrTrefor it's also key for analyzing the RZ function and deriving the prime number theorem
@riadsouissi
@riadsouissi Год назад
Though each sum of 1/np^n converges when n is larger than 1, the infinite sum of these sums does not necessarily converge (it does but this needs to be proven rather than just stated)
@changyauchen
@changyauchen Год назад
This proof is quite elementary and easy compared to other proofs I have seen. I had pondered this problem but never find an answer myself, this proof is the most satisfying one.
@jacoblehrer4198
@jacoblehrer4198 Год назад
Elementary*
@alphalunamare
@alphalunamare Год назад
@@jacoblehrer4198 I never met Watson but are you implying that any polynomial in the set under question can be reformulated with prime exponents only?
@hassanakhtar7874
@hassanakhtar7874 Год назад
Agreed, it is comparable to the proof provided by Erdös. This one requires minimal facts about primes and more series manipulation, while the Erdös proof has minimal series manipulation but translates the problem to a counting one.
@ronald3836
@ronald3836 10 месяцев назад
It seems to be by James A. Clarkson from 1966. Wikipedia has a link to it. en.wikipedia.org/wiki/Divergence_of_the_sum_of_the_reciprocals_of_the_primes
@eofirdavid
@eofirdavid Год назад
Nice proof. Heard of similar proofs, but never this one. Just wanted to add that it is very "natural" to try to move from a problem with prime numbers to a problem with integers, since (1) the integers are generated by the primes, and (2) it has a much better structure which we can use (it has addition and multiplication) that the primes don't have. The next step would be to move from the integers to the real line, which has an even better structure - we have continuity, differentials, integrals, and all of the rest of the tools from analysis. For example, this is usually how you show that the Harmonic series diverges, by comparing it to the integral of 1/x. We can also do it on the other direction, for example trying to show that the sum of reciprocals of primes which are 1 mod 4 diverge to infinity, by moving to the same problem with all of the primes. In particular, this leads to one of Dirichlet's theorems that shows that there are infinitely many primes which are m mod n whenever gcd(m,n)=1.
@johnchessant3012
@johnchessant3012 Год назад
According to the Müntz-Szász theorem, this implies that every continuous function on [0, 1] is the uniform limit of a sequence of polynomials where all the exponents in all the polynomials are prime numbers.
@cblpu5575
@cblpu5575 Год назад
There was a man long ago who did work on the reciprocals of primes. He saw after how long the decimal digits of the reciprocals of primes repeated and he calculated them upto many thousands of primes. When his book was inspected, you see some corrections he made and they are either halving or doubling the original number that he had entered. He clearly had some formula for calculating them because if they were counting mistakes the corrections would be off by 1 or 2 or some other small number but he made corrections that showed he knew some sort of formula for the number of digits after which the decimal digits of the reciprocals of primes repeat. I saw this in a numberphile video and was curious. Thus video reminded me of that. Does anyone know more information about this?
@maynardtrendle820
@maynardtrendle820 Год назад
It's about William Shanks, and it's a Numberphile video called 'The reciprocals of primes'. It's a Matt Parker episode.
@Alex_Deam
@Alex_Deam Год назад
I remember that Numberphile vid. Here's the rough idea: Let's set 1/p = 0.[n][n][n]... where [n] is some repeating string of digits. And let's say [n] is A digits long. That means we can multiply by 10^A and get: (10^A)/p = [n].[n][n][n]... = [n] + (1/p) Multiply LHS and extreme RHS by p: 10^A = p[n] + 1 Remember, [n] is just some integer. So the RHS is just one greater than a multiple of p. So we can reduce mod p: 10^A =(congruent) 1 (mod p) Now, Fermat's Little Theorem tells us that 10^(p-1) =(congruent) 1 (mod p), so long as gcd(10,p)=1 (which it will be for p =/= 2 or 5). From this it can be quickly seen that A must divide (p-1). You can also just say the same thing using Lagrange's Theorem from group theory instead. (I can expand on the Fermat/Lagrange step if you want, this is just the summary of the argument) Anyway, once you know that A must divide (p-1) you have much fewer numbers to check, and there are number theory shortcuts that can make those calculations (mod p) faster. And presumably this explains how he was out by a factor of two sometimes - 2 will always be a factor of (p-1) for the primes we're interested in, and (10^A)^2 = 10^(2A) (or vice versa) hence easy tog et a factor of two, remembering that (+/-1)^2 = 1.
@shruggzdastr8-facedclown
@shruggzdastr8-facedclown Год назад
I saw the same Numberphile video -- thought it was a Pi Day-related piece, but my memory might be faulty, thereof.
@muriloporfirio7853
@muriloporfirio7853 Год назад
ru-vid.com/video/%D0%B2%D0%B8%D0%B4%D0%B5%D0%BE-DmfxIhmGPP4.html
@quantumgaming9180
@quantumgaming9180 Год назад
@@Alex_Deam wtf. Did you really made this explanation by yourself or did you seen it somewhere else?
@angelmendez-rivera351
@angelmendez-rivera351 Год назад
My favorite proof of the divergence of this series utilizes the prime number theorem. The prime number theorem states that the prime counting function π satisfies the property that lim π(m)/(m/ln(m)) (m -> ∞) = 1. What this implies is that the mth prime number p(m) satisfies lim p(m)/(m·ln(m)) (m -> ∞) = 1. Therefore, the sequence of partial sums of 1/p(m) is asymptotic to the sequence of partial sums of 1/(m·ln(m)). Now, we can use the integral comparison test, noting that that the integral on [e, x] of 1/(t·log(t)) is bounded from above by the sequence of partial sums of 1/(m·ln(m)). But, hey, wait a minute! The integral can actually be evaluated explicitly, and it is equal to ln(ln(x)). But, look, lim ln(ln(x)) (x -> ∞) = ∞ !! Therefore, the integral diverges, so the sequence of partial sums of 1/(m·ln(m)) diverges, so the series of reciprocal prime numbers diverges. Q. E. D.
@zitengwang136
@zitengwang136 Год назад
Finally getting someone is actually knowing math and teaching math properly on RU-vid.
@robharwood3538
@robharwood3538 Год назад
@2:29: Should be: If |x| < 1, then geometric series converges. Counterexample to video's statement: If x = -2, then x < 1, but the geometric series does not converge.
@DrTrefor
@DrTrefor Год назад
Quite right! I only had positive terms in all my series so I want thinking about negatives at all!
@hareal3904
@hareal3904 Год назад
9:00 here's a more rigorous proof: Set p1 * p2 * .... * pn = k 1 / (1 + ik) > 1 / (k + ik) = (1/k)(1/ (1 + i)) Since k > 1 most definitely, we made the denominator bigger so the entire term is smaller Sum i from 1 to inf of 1 / (1 + ik) > Sum i from 1 to inf of (1/k)(1 / (1+i)) = (1/k)(Sum i from 1 to inf of 1 / (1+i)) = (1/k)(Sum of harmonic series - 1) = ♾️
@MasterHigure
@MasterHigure Год назад
I made a video covering Erdös' proof of this claim a couple of months ago, and I thought that proof was elegant enough (he likewise splits the series into two, but then uses that to show that there are fewer than N natural numbers in {1, 2, ..., N} for some large N, which is a contradiction). Why have I not seen this one before? It's wonderful! Where / when / who is it from? Little side note: You don't need contradiction here. You've really just shown that for any N, we get X>1. There is no actual need to ever assume X
@victorhermestorrestomara3050
Nice proof! This made me think about how this series converges even more slowly than the harmonic series and wether there's a way to construct the slowest converging series possible or maybe an iterative method for log(log(log...(N)...)) converging series
@DrTrefor
@DrTrefor Год назад
right?!
@TheKudo555
@TheKudo555 Год назад
I had thought about this some while ago, and concluded that it wasn't possible. Suppose there was a sequence u_n such that : - u_n is positive and increasing - u_n diverges (u_n -> + inf) - for all positive divergent sequence v_n (v_n -> +inf), there is a > 0 such that for all n : v_n / u_n > a. For simplicity, I will use the shorthand u_x for a positive real x defined as : u_x = (x - Floor(x)) u_Floor(x) + (Floor(x + 1) -x) u_Floor(x+1). Consider now the sequence w_n = u_(u_n). We have Lim [n -> +inf] w_n / u_n = Lim [x -> +inf] u_x / x. Using the third point with v_n = log(n), we have a positive a>0, st u_n x] (1/x) * Product [k : 1 -> K] 1/log^k(u) = log^(K+1) (x) For example : Int [u : 1 -> x] 1/u log(u) log(log(u)) = log(log(log(x))) Considering the series z_n = Sum [p : 1 -> n] (1/p) Product [k : 1 -> K] 1/log^k(p), A series integral comparison would probably conclude that z_n is equivalent to log^(K+1)(n).
@angelmendez-rivera351
@angelmendez-rivera351 Год назад
There is no such a thing as the slowest diverging series, because there is no such as the slowest growing sequence.
@ronald3836
@ronald3836 10 месяцев назад
It would be interesting to see a similar "prime-based" series that grows like log(log(log(N))). Perhaps sum_i 1/p_p_i ? (Probably does not work.)
@F-S.
@F-S. Год назад
Thank you! I knew that this series converges but never saw a proof. A suggestion: Can you make a simular video (for us non-mathematicans) about the series of the rezipricals of the sum of the twin primes?
@unvergebeneid
@unvergebeneid Год назад
*diverges
@DrTrefor
@DrTrefor Год назад
Turns out reciprocal twin prime series converges!
@Jack_Callcott_AU
@Jack_Callcott_AU Год назад
Thank you Dr Bazett , I really enjoyed this discussion and proof. It's interesting to think about series that are on he borderline of convergence/divergence. It seems intuitive that the reciprocal of primes might converge, because primes are sparse compared to integers for large numbers .
@jerry5149
@jerry5149 4 месяца назад
Maple is an incredible tool, I hope someday every child will use it!
@diniaadil6154
@diniaadil6154 Год назад
Nice proof. Fun fact: this series diverges even slower than the harmonic series (equivalent to log(log(p_n))
@DrTrefor
@DrTrefor Год назад
Isn’t that cool?
@ronald3836
@ronald3836 10 месяцев назад
Shouldn´t that be log(log(n)), even worse?
@ronald3836
@ronald3836 10 месяцев назад
No I'm wrong. Or better: you are right.
@andrewharrison8436
@andrewharrison8436 Год назад
Second time through it made sense. So to prove the series diverges you throw away the early terms, create a new series then show that if you throw away enough terms from that you get a series that you recognise as divergent. Yes, that makes sense.
@sz7232
@sz7232 Год назад
Your explanation is so clear, thanks!
@unvergebeneid
@unvergebeneid Год назад
In the beginning I was kinda hoping the series would converge. Would've been cool if there was a number that's the sum of the reciprocals of all the primes. But I don't want to break reality, so this is fine, too.
@DrTrefor
@DrTrefor Год назад
haha "wishful mathematics" is my favourite:D
@johnchessant3012
@johnchessant3012 Год назад
this suggests a rather funny "proof": if the series converged to some constant, that constant would be famous enough that I would've heard of it. therefore it must diverge :)
@geoperry
@geoperry Год назад
1 / ( 1 - r ) for 0 < r < 1 will be greater than one for r = 1/2 1/2 + 1/4 + 1/8 ... is not greater than one let sigma for geometric series shown at 2m34s commence with i=0, or let sum be ( 1 / ( 1 - r ) ) - 1 ... superfluous outer parentheses added for emphasis
@considerthehumbleworm
@considerthehumbleworm Год назад
DUDE. At 5 minutes in when I realized that the new series contained every (ish) integer, my jaw genuinely dropped. cool fuckin proof
@meurdesoifphilippe5405
@meurdesoifphilippe5405 Год назад
I guess it would have been possible to consider the series of 1/(-1+ i* p_1...p_N) instead which is greater than 1/(p_1...p_N) * harmonic series, hence diverges without comparison argument.
@rosskrt
@rosskrt Год назад
That hippopotenuse shirt is fire
@ffc1a28c7
@ffc1a28c7 Год назад
This can be done a bit more powerfully to show that there are infinitely many primes. When you use the fact that 1/(1+p1p2..pn) has only prime factors above pn, you are implicitly using the fundamental theorem of arithmetic which is proven using the fact there are infinite primes.
@ronald3836
@ronald3836 10 месяцев назад
I don't think that uses the fundamental theorem of arithmetic. In its simplest form, unique factorization means "p | ab => p | a OR p | b" for all irreducible numbers p and integers a,b, and this we never use. If you factorize (not necessarily uniquely) A = 1 + k * p_1 * ... * p_N into irreducible numbers A = q_1 * ... * q_M, then it is clear that none of the q_j can be a p_i, since A mod q_j = 0 for all j and A mod p_i = 1 for all i. So any number A_k of this form is the product of irreducibles q_j > p_N (perhaps in more than one way), which means 1/A occurs in X+X^2+X^3+... (perhaps multiple times). So X+X^2+... >= sum 1/A_k = inf.
@mst7155
@mst7155 Год назад
For those of us who aren't calculus professors there is no need for any" limit comparison test". Otherwise the proof is extremely beautiful and simple!!!!!! But this doesn't mean is not inventive! Thank you a lot for this new proof.
@michaelleue7594
@michaelleue7594 Год назад
So, say that you have a sequence consisting of the sums of the first n terms of a sequence like (1/2,1/(2*3),(1/(2*3*5),...,1/p_n#) and then subtract the results from 1. So like (1/2, 1-1/2-1/(2*3), 1-1/2-1/(2*3)-1/(2*3*5),...,1-1/2-1/(2*3)-1/(2*3*5)-...-1/(p_n#)). This sequence converges to 0 per the Prime Number Theorem. Is there any way to describe the behavior of this sequence at finite positions (as opposed to at n=infinity)?
@complexplane6756
@complexplane6756 Год назад
Very nice video. My only suggestion is to mention we are dealing with series which monotonically converge. This helps with notions of convergence of subsequences.
@Mutual_Information
@Mutual_Information 7 месяцев назад
Wow really enjoyed this
@ivailogeorgiev1389
@ivailogeorgiev1389 Год назад
What about the sum of reciprocal of the square of primes- it definitely converges because it's smaller than Basel problem which we know it's equal to (pi^2)/6. It will be interesting to see to what it converges.
@jamesknapp64
@jamesknapp64 Год назад
I would just use direct comparison as 1/(1 + i × prime product) > 1/(2 × i × prime product) = 1/2 × 1/(prime products) × 1/i Also Sum 1/i diverages is just showing its bigger than 1 + 1/2 + 1/4 + 1/4 + 1/8 + 1/8 + 1/8 + 1/8 +.... Which will diverage to Infinity. Thus this can be shown with these arguements to College Algebra or advanced Int Alg (High Schoolers) students without any calculus ideas.
@crazyspider17
@crazyspider17 Год назад
how is this proof not more famous, it's so simple
@mjp121
@mjp121 10 месяцев назад
Your end claim appears to work, but only because both series contain only positive numbers- it’s relatively easy to imagine a convergent series containing a divergent subseries if the convergent series contains a term (-1)^i (or sin(i) if we feel fancy) A well known example would be sum i = 1 to inf {((-1)^i-1)/i = ln2 We can recognize the shifted geometric series 1+1/3+1/5+… is a subset of the ln(2) summation, diverges to positive infinity, and is at all points greater than the sum of prime reciprocals, yet the parent summation converges. I would need further consideration on whether or not similar slight of hand could be used for a series of positive values can converge as a subseries diverges, and my instinct is to say it cannot (an infinite positive term will always overwhelm a finite term in the limit) but I’d be curious if mathematicians in comments can prove my instinct wrong
@yumnuska
@yumnuska Год назад
I really enjoyed this video. I almost passed it by because of the “shocked” click bait thumbnail. It was truly the title that convinced me to watch the start, and I was pleasantly surprised to see that you’ve got a semi-Mathlogger style for a topic I’d not thought about before. But normally I skip the shocked pikachu thumbnails. I haven’t subbed yet.
@DrTrefor
@DrTrefor Год назад
Ha. To be honest, I find them a bit cringe myself. But the analytics is overwhelmingly clear that when I give those types of thumbnails the click though rate is just higher. It's a wierd phenomenon
@yumnuska
@yumnuska Год назад
@@DrTrefor that’s fair. I get that the algorithm can encourage creators to encourage creators to “work it”. I really loved your video. I’ve subbed, since I’d rather see more of your style than less. I hope you get big enough that you don’t need to shock me every time!! 😂
@georgesinger9227
@georgesinger9227 Год назад
Where do you get the math t-shirts? What app do you use on iPad to write expressions with Apple Pencil?
@DrTrefor
@DrTrefor Год назад
Merch link in description! I mainly use OneNote
@Chalisque
@Chalisque Год назад
Summary: Assume sum(1/p_n,n=1..) converges. Then for some N, X = sum(1/p_n,n=N..) < 1. Then consider sum(X^n,n=1..). Since X < 1, this sum converges. But consider sum(1/(1+ip_1p_2...0_n),i=1..) Clearly sum(X^n,n=1..) > sum(1/(1+ip_1p_2...0_n),i=1..) as all terms are non-negative and every term in the rhs turns up in the lhs somewhere. But lim((1/(1+ip_1p_2..p_n))/(1/i),i=1..) = lim(i/(1+ip_1p_2..p_n)) = lim(i/ip_1p_2..p_n)=1/p_1..p_n is finite and nonzero. So since sum(1/i,i=1..) diverges, so does sum(1/(1+ip_1..p_n),i=1..), and thus so does sum(X^n,n=1..) which is a contradiction.
@Chalisque
@Chalisque Год назад
The first 'clever bit' is remembering the ε-N definition* of a convergent series, and then that if 0
@esteger1
@esteger1 Год назад
I really liked this proof. Thanks!
@saggetarious97
@saggetarious97 Год назад
At 2:34 the geometric series should start from i=0 for it to be 1/1-x , right? Very interesting video, thanks for all the great content! 😁
@GeoffryGifari
@GeoffryGifari Год назад
huh... have mathematicians studied the properties of similar objects where summation is done over all primes? i'm thinking (where p goes over all primes) Σ xᵖ Σ xᵖ/p! (maclaurin series of eˣ, but just the primes) Σ 1/xᵖ (does this converge? closed form?) Σ 1/p² (this should converge... right?)
@xanthoconite4904
@xanthoconite4904 6 месяцев назад
Now I would love to know what the alternating sum of inverse primes converges to.
@adamlindstrom5750
@adamlindstrom5750 Год назад
Very neat proof. Although it did not need to be framed as a proof by contradiction. What you are showing is that every tail of the reciprocal of primes series has a sum X greater than 1, which you can show directly by, as in the video, showing that the geometric series X + X^2 + ... diverges. This is essentially the same proof but rewritten as a direct proof instead, which imo almost always makes things cleaner and clearer.
@DrTrefor
@DrTrefor Год назад
True. Actually this is quite thematic of lots of contradiction proofs that it is more a choice of presentation style than an actual requirement
@jack_evoniuk
@jack_evoniuk Год назад
1:49 that's the funniest summation sign I've ever seen.
@DrTrefor
@DrTrefor Год назад
Microsoft PowerPoints math font is simply terrible!
@Noissimsarm
@Noissimsarm Год назад
Does this only work because all the terms are positive? The series 1/2 -1/3 +1/4 -1/5 + 1/6... converges (not absolutely) but the subseries 1/2 +1/4 +1/6 .... is divergent. So it's not necessarily true that if the subseries diverges then the main series diverges as well.
@DrTrefor
@DrTrefor Год назад
Correct, only for positives. We are using comparison test here and yes with negatives you can’t set it up the same way.
@andrewwalker7276
@andrewwalker7276 9 месяцев назад
Great video! There are a number of videos out there on the divergence of the harmonic series and the reciprocals of the prime numbers. Maybe there are some on generalising the harmonic series to the Riemann zeta function. The reciprocal of prime numbers can similarly be generalised to the prime zeta function. I have never seen a video on this! If this is something of interest, I can send a link to the Carl Froberg paper on the prime zeta function. It also has a wikipedia page, and I have been calculating its zeros for many years!
@Ekaterina.Kurkina
@Ekaterina.Kurkina Год назад
Good afternoon! Very interesting video! You speak very well! I am glad to welcome you, colleague!
@DrTrefor
@DrTrefor Год назад
Thank you so much!
@sr.tarsaimsingh9294
@sr.tarsaimsingh9294 Год назад
@Ekaterina Kurkina You also get one more subscriber due to this comment. Sir, Thanks to yours explanation and efforts ❤.
@maniman121
@maniman121 Год назад
I just saw an interesting conversation about this on twitter this week
@bilalabbad7954
@bilalabbad7954 Год назад
Thanks
@ruferd
@ruferd Год назад
Isn't there some interpretation here that since 1/n^2 converges and 1/p diverges, there are "more primes" than squares? (Or at least, the primes occur more frequently than squares)
@DrTrefor
@DrTrefor Год назад
I believe there is a prime between amy consecutive squares!
@angelmendez-rivera351
@angelmendez-rivera351 Год назад
@@DrTrefor I think the concept he is alluding to is the concept of sparseness. Consider S to be some subset of N, and let X : N -> S be a bijective sequence of numbers in S. S is called a sparse set of N if the sequence of partial sums of 1/X converges. In other words, T = {n in N : m^2 = n, m in N} is a sparse set, since ζ(2) is finite, but P = {n in N : n is prime} is not a sparse set. Since geometric series converge, the set of powers of any given natural number is a sparse set. Etc.
@jondohnson8417
@jondohnson8417 Месяц назад
2:33 isn't that formula for the geometric series wrong? Maybe i=0 would be correct
@FrankHarwald
@FrankHarwald Год назад
1:50 I love how your capital SIGMA looks like a pine tree :D
@gavinwalsh9860
@gavinwalsh9860 3 месяца назад
1:48 I love your videos, and I admire your teaching style, but we need to talk about the way you draw sigmas by hand.
@the_real_glabnurb
@the_real_glabnurb Год назад
I think one critical part of the proof has been left out: @8:00 the claim is that it's a subseries of the geometric series, but no proof has been given.
@DrTrefor
@DrTrefor Год назад
This was largely proved prior to the statement. That is I was arguing that the terms in the subseries would all look like the terms in the geometric series.
@michaelperrone3867
@michaelperrone3867 Год назад
Beautiful proof!
@Leloup7
@Leloup7 Год назад
Is there a relationship between the reciprocal serie of diverging primes and the fact that the set of primes is infinite? why are we interested in the divergence of this serie?
@benoitalain5833
@benoitalain5833 Год назад
Perhaps a simpler proof: Notice that p_i progresses like i*ln(i), and Sum(1/(i*log(i))) diverges for any log. We can check that that sum of 1/(i*log_2(i)) diverges by using the same trick as for the harmonic series. Group the first 2 terms, the next 4, the next 8, 16, 32, etc. In the harmonic series each group is >= 1/2 so the sum of the groups is infinite. In the 1/(i*log(i)) series, up to some multiplicative constant the first group is >= 1, the second is >= 1/2, the third is >= 1/3, etc. The sum of all the groups follows the harmonic series which diverges, so the 1/(i*log(i)) series diverges too.
@ronald3836
@ronald3836 10 месяцев назад
Starting with the prime number theorem is not really simpler :)
@ofekn
@ofekn Год назад
It's a bit counter intuitive, because the chance of a number k to be a prime is 1/k so i would expect the sum to be similar to 1/k^2 which converges.
@koenth2359
@koenth2359 Год назад
That is what I initially thought too, but the chance is not 1/k, but rather 1/ln(k). Source: wikipedia (page 'prime number theorem') . Also see my comment (of a few minutes ago) to the video, where I do the integral.
@ofekn
@ofekn Год назад
@@koenth2359 Oh, i see, you are right. Probably got confused because I learned it from the computer science perspective and there you are using the length of the number rather than the number itself.
@mathisnotforthefaintofheart
The series would be convergent if alternating. Is that sum known?
@DrTrefor
@DrTrefor Год назад
Certainly converges by alternating series test, so the number can be approximated but I don't know if it is famous enough to be given a fancy name.
@tokajileo5928
@tokajileo5928 Год назад
what about the reciprocal twin primes series?
@DrTrefor
@DrTrefor Год назад
We don’t even know it is infinite!
@RSLT
@RSLT Год назад
Fantastic Video. Informative and Cool approach!
@DrTrefor
@DrTrefor Год назад
Glad you liked it!
@IvaHaze
@IvaHaze Год назад
You could assume that since the reciprocal primes series is just a "portion" of the harmonic series (which is divergent), then the reciprocal primes series is divergent.
@DrTrefor
@DrTrefor Год назад
1/n^2 is just a portion of the harmonic series too, but it converges!
@IvaHaze
@IvaHaze Год назад
@@DrTrefor true
@ronald3836
@ronald3836 10 месяцев назад
@@DrTrefor If 0 < a_1 < a_2 < ... and sum 1/a_i = inf, can we show that sum 1/a_p_i = inf?
@feynstein1004
@feynstein1004 Год назад
I was wondering about this recently. Too bad the primes diverge. I was hoping to reverse engineer a formula for them using the reciprocal sum 😅
@alphalunamare
@alphalunamare Год назад
An algorithm to determine the nth prime number does exist ... one just has to be patient.
@feynstein1004
@feynstein1004 Год назад
@@alphalunamare Algorithm =/= formula. The algorithm you're talking about is a clever hack of sorts.
@hiredfiredtired
@hiredfiredtired Год назад
@@feynstein1004 willians formula for primes be like
@angelmendez-rivera351
@angelmendez-rivera351 Год назад
@@feynstein1004 There are dozens of formulae characterizing the prime numbers out there. You need to try searching harder.
@feynstein1004
@feynstein1004 Год назад
@@angelmendez-rivera351 There are? 🤔
@disgruntledtoons
@disgruntledtoons Год назад
You can probably prove that for any convergent geometric series, the elements of that series, after a certain point, are always less than the corresponding elements of the reciprocal prime series.
@koenth2359
@koenth2359 Год назад
Hmm, will need a few more view to digest it all, very nice. But I would have thought so, because we could approximate the result for large n by multiplying 1/n with the prime density function (which converges to 1/ln(n) for large n). If this sum is finite, the following integral should be finite too (I know, sloppy notation): ∞ ∫ 1/(x ln x) dx = ln(ln(∞)) - ln(ln(x0)) = ∞ x0
@paradoxicallyexcellent5138
@paradoxicallyexcellent5138 Год назад
As with most (if not all?) proofs by contradiction, you didn't need to use contradiction. "Some series diverges and is a subseries of some geometric series with ratio X, so X >= 1. But X is an arbitrary tail of the reciprocal prime series so the reciprocal prime series diverges."
@DrTrefor
@DrTrefor Год назад
Very true, often just stylistic differences the core is the divergence sub series of a convergence series
@ronald3836
@ronald3836 10 месяцев назад
Don't you need contradiction to go from "arbitrary tail is >=1" to "the series converges"? (I define "series converges" as "for every M there is an initial part that exceeds M".) I guess one can use X>=1 to create disjoint initial parts of the series that are all >= 1/2, and when you take [M/2] + 1 of them, you can sum them to get >=M. But I personally prefer math without this hassle ;-) (even though I am a compatriot of LEJ Brouwer, and I am afraid no other Dutch mathematician compares to him).
@paradoxicallyexcellent5138
@paradoxicallyexcellent5138 10 месяцев назад
​@ronald3836 Show me any proof that proceeds "Assume B is not true and A is true... something something something... contradiction" and I will show you a proof "Suppose B is not true... something something something... therefore A is not true." It's not more hassle for me, except sometimes it forces me to understand things better. Because when I do a proof by contradiction, I have the sensation that I flailed around with symbols in Leprechaun and Unicorn-land until some contradiction fell on my lap, and I feel unenlightened by that exercise.
@ronald3836
@ronald3836 10 месяцев назад
@@paradoxicallyexcellent5138 How do you prove the non-countability of the reals? Or do you simply not accept the statement that the reals are not countable?
@paradoxicallyexcellent5138
@paradoxicallyexcellent5138 10 месяцев назад
@@ronald3836 Let f be any function from naturals to reals... something something diagonal... therefore f is not onto. At no point do I have to assume f is onto to prove it is not onto. This is merely a direct proof that "For all functions f from N to R, f is not onto." Assuming it is onto "for contradiction" is a farce.
@wilurbean
@wilurbean Год назад
I'm in junior level "theoretical methods" which is a math survey class at my university. We just covered series and how they can make/solve special functions like legendre, gamma, asymtopic series, etc Anyways, study group working on hw and we found a version of the p-series test where we would compare 1/f(n) whatever that is to 1/n^p where we're taking the limit at p-> 1+ So any value just to the right (greater) of p=1 and comparing. Does this have a name or anything? It's been very useful
@DrTrefor
@DrTrefor Год назад
I think just “p test” is most common
@alphalunamare
@alphalunamare Год назад
7:50 say ... when you 'claimed' I went into a confused loop. Did you mean: "I will show that" ? I only ask because the flow of understanding is easier without tangential inputs. I mean, it was the first time you had mentioned 'any' claim in the first place, so it demanded a rerun of the video up until that point as a minimum.
@raseelab4475
@raseelab4475 Год назад
Hello professor , Are going to continue with the money math series ?
@danielr2040
@danielr2040 Год назад
Interestingly, the sum of the reciprocals of the twin primes converges.
@DrTrefor
@DrTrefor Год назад
I suppose this gives some hints as to the rarity of twin primes in the distribution
@MyOneFiftiethOfADollar
@MyOneFiftiethOfADollar Год назад
The sum of twin primes is always divisible by 12 p>3 ,eg 41+43=84 = 12x7. Would think that would be used in proving the the convergence 1/3 + 1/5 + 1/5 + 1/7 + 1/11 + 1/13 + ….
@cufflink44
@cufflink44 Год назад
Very interesting indeed. What does it converge to? And does that constant have a name?
@danielr2040
@danielr2040 Год назад
@@cufflink44 Yes, Brun’s constant. en.m.wikipedia.org/wiki/Brun%27s_theorem
@cufflink44
@cufflink44 Год назад
@@danielr2040 Beautiful. THANK YOU!
@LP-zz8wo
@LP-zz8wo Год назад
I want to ask you about analytic geomtry Do they teach it as a entire subject Or it is part of another subject or what?
@leonhardeuler675
@leonhardeuler675 Год назад
I disagree with the stipulation that the sums of the reciprocal primes is greater than the sum of the reciprocals of powers of 2. I don't think you've proved this here.1:00
@DrTrefor
@DrTrefor Год назад
I’m effectively citing the fact there is always a prime between n and 2n, but just illustrating that for the first few terms as that isn’t the proof I’m aiming for in this video.
@leonhardeuler675
@leonhardeuler675 Год назад
@@DrTrefor I think that would have been interesting in its own right. Given it's already an over 10 minute video, I don't see the need to skip it. For those of us in the know, the devil in infinite series is always in the detail and when you skip it, I don't think "that must have been boring". I think "why did he skip it?" "what's he hiding?". More importantly, I think you should be careful about the motivation for your content. Instead of curating maths, deciding what the public gets to see (and what it can't), consider instead that your content should be about you being a mathematician and showing the world what that is like and how you see things. Afterall, you're not a gatekeeper. You own none of this. That is the difference between Matt Parker and 3b1b and the rest of the horde of maths channels out there.
@mst7155
@mst7155 Год назад
This is a very beautiful ❤️❤️❤️❤️❤️❤️❤️❤️❤️❤️❤️❤️❤️❤️❤️ proof! Thank you so much!!!!!!!
@cxpKSip
@cxpKSip Год назад
6:14 My head is screaming just to use the Fundamental Theorem of Arithmetic and bypass this argument directly.
@benjaminshropshire2900
@benjaminshropshire2900 Год назад
I think there is a skipped step at the end: what is actual proven is that any suffix of the original series is greater than 1. To be pedantic, you still need to prove that it's not a finite term greater than 1. (Intuition tells me that follows, but using intuition with infinities is a risky bet.) Also, why prove by contradiction? Wouldn't the argument work just as well if you start by proving that all the generated sub-series diverges, then show that implies that X is always grater than 1 for every possible suffix and conclude by showing that implies the original series diverges?
@apburner1
@apburner1 Год назад
This does not require a formal proof. There are an infinite number of primes, therefore no matter how small the reciprocal becomes there are infinite more to add to the sum.
@simohayha6031
@simohayha6031 2 месяца назад
That logic doesnt hold up all the time. The reciprocals of the squares, also has infinite terms but it converges
@mienzillaz
@mienzillaz Год назад
Can you prove using same tools that series that converges and we know it by other methods truly converges?
@frederic0chopin
@frederic0chopin Год назад
1:49 omg bro that sigma is cursed
@davidgillies620
@davidgillies620 Год назад
Essentially the fact that the sum of the reciprocals of the primes is divergent is yet another proof that there is an infinite number of them.
@DrTrefor
@DrTrefor Год назад
The object 1+p_1....p_n I used in this proof is actually the same object that can be used to show infinitely many primes!
@vascomanteigas9433
@vascomanteigas9433 Год назад
The sum of reciprocals of twin primes converges, defining the Braun constant. If Braun Constant are an irrational Number, then exists infinite twin primes.
@maths1993
@maths1993 Год назад
Excellent ❤️, Could you tell us about the software you used to do this video and equations. Thank you
@DrTrefor
@DrTrefor Год назад
It’s just PowerPoint in the background!
@julianfogel5635
@julianfogel5635 Год назад
Enjoyed. What about series made up of every k'th prime? For which values of k will they converge if any? You showed it diverges for k = 1, but what about k=2, the sum of the reciprocals of every second prime?
@DrTrefor
@DrTrefor Год назад
Oh fun, that's a nice puzzle, how much can you remove until you get something convergent!
@normanstevens4924
@normanstevens4924 Год назад
@@DrTrefor Sum of reciprocals of n'th primes never converges. Assume it does then we can generate a new series by adding the reciprocals of the next primes, i.e. those where p = 1 (mod n). This sum is less because all the denominators are larger. Similarly for the sum of all primes where p = a (mod n) for 0
@sk8erJG95
@sk8erJG95 Год назад
Don't know why my other reply didn't pop up, but the key to all this sum-of-reciprocal stuff is the Bloom-Sisask theorem, which (in a sense) says that the sum of reciprocals of elements of A converges if the density of A is C/(logx)^(1+e) for some C>0 and e>0. As the density of primes is 1/logx, this tells us the sum of reciprocals of primes diverges. The subset of kth primes would have density porportional to the density of primes, which is x/logx, so that e mentioned in the theorem above will be 0, and the sum will still diverge. The theorem is actually about arithmetic progressions in A, but you can translate.
@julianfogel5635
@julianfogel5635 Год назад
@@sk8erJG95 Had to look up density of a set of reals: en.wikipedia.org/wiki/Natural_density. I found a 2018 paper by Bloom and Sisak on arxiv but it's beyond my comprehension.
@sk8erJG95
@sk8erJG95 Год назад
@@julianfogel5635 Look at Thomas Bloom's research page, he posts links to his papers and a small summary of the results of those paper. His summary: "We show that if A = {1,...,n} contains no 3-term arithmetic progression, then |A|
@nakhleasmar9175
@nakhleasmar9175 Год назад
Nice proof.
@stighemmer
@stighemmer Год назад
Beautiful!😊
@Cjendjsidj
@Cjendjsidj Год назад
Wow this is so cool
@DestroManiak
@DestroManiak Год назад
The fact that this series diverges is DEEPLY, PROFOUNDLY UNSETTLING. It feels soooooo wrong.
@DrTrefor
@DrTrefor Год назад
Haha I know the feeling!
@ddystopia8091
@ddystopia8091 Год назад
Why do geometric series is lower than 1/p_i? I think it would be harder to proof then that from video :D
@twixerclawford
@twixerclawford Год назад
I wonder what the largest of the series of naturals (primes or otherwise) that converges is. I imagine you can get arbitrarily large, but what about restricting it to "relatively" simple patterns? That might be a really interesting question
@DrTrefor
@DrTrefor Год назад
I guess it depends what you mean by simple!
@HarshitBujarBaruah
@HarshitBujarBaruah Год назад
Amazing
@ccg8803
@ccg8803 Год назад
Great video
@DrTrefor
@DrTrefor Год назад
Thanks!
@Siirxe
@Siirxe Год назад
1:9 not the geometric series
@jake967
@jake967 Год назад
+
@jessstuart7495
@jessstuart7495 Год назад
Euler would approve.
@vampire_catgirl
@vampire_catgirl Год назад
The way you draw Sigma hurts to look at
@CPep
@CPep Год назад
I thought I saw a 1/14 on the thumbnail
@janvdplaat3067
@janvdplaat3067 Год назад
This vlog is made for mathematicians who have a degree in algebra. .
@deserado11
@deserado11 Год назад
... say what? ...
@andyparadis342
@andyparadis342 Год назад
Your thumbnail, you're smarter than that!?
@CarlosFloresP
@CarlosFloresP Год назад
Love the vid but proofs are 💤💤💤
@DrTrefor
@DrTrefor Год назад
haha fair but I love them:D
@CarlosFloresP
@CarlosFloresP Год назад
@@DrTrefor proofs are fun but they are usually hard. That's why I don't like them lol
@vascomanteigas9433
@vascomanteigas9433 Год назад
So far, One of my laborious Proof was using keyhole contour complex integral to prove the Riemann Zeta Functional equation.
@robertcotton8481
@robertcotton8481 Год назад
Hate education turning into ad city
@user-eq6te1mw8e
@user-eq6te1mw8e Год назад
No ? Calculus is actually usefull irl . Prime numbers is just something only phreaking pure math people care and it actualy has zero aplications
@DrTrefor
@DrTrefor Год назад
All our encryption is based on prime numbers!
@indukumarigupta5039
@indukumarigupta5039 Год назад
Sir bring Indian Olympiad questions
@johnnicholson8811
@johnnicholson8811 Год назад
I do wish when someone talks about this they would say something on Meissel-Mertens constant. en.wikipedia.org/wiki/Meissel%E2%80%93Mertens_constant
@DrTrefor
@DrTrefor Год назад
Didn't make it in this video, but absolutely that is part of the larger conversation
@johnnicholson8811
@johnnicholson8811 Год назад
@@DrTrefor Well, have you talked about the OEIS constants at A350581 or A350582? Would love to hear what you think about them. I find the difference between a bit weird. The question on my mind is what does this imply?
Далее
Topology is weird: The Ham Sandwich Theorem
12:37
Просмотров 45 тыс.
Мой телеграмм: v1ann
00:14
Просмотров 51 тыс.
The Reciprocals of Primes - Numberphile
15:31
Просмотров 1,6 млн
The Fastest Multiplication Algorithm
13:58
Просмотров 114 тыс.
What is a TENSOR? (Really this time!)
59:24
Просмотров 14 тыс.
An Exact Formula for the Primes: Willans' Formula
14:47
The Bernoulli Integral is ridiculous
10:00
Просмотров 702 тыс.